Difference between revisions of "2020 AMC 12B Problems/Problem 14"

(See Also)
(Redirected page to 2020 AMC 10B Problems/Problem 16)
(Tag: New redirect)
 
Line 1: Line 1:
==Problem==
+
#REDIRECT [[2020 AMC 10B Problems/Problem 16]]
 
 
Bela and Jenn play the following game on the closed interval <math>[0, n]</math> of the real number line, where <math>n</math> is a fixed integer greater than <math>4</math>. They take turns playing, with Bela going first. At his first turn, Bela chooses any real number in the interval <math>[0, n]</math>. Thereafter, the player whose turn it is chooses a real number that is more than one unit away from all numbers previously chosen by either player. A player unable to choose such a number loses. Using optimal strategy, which player will win the game?
 
 
 
<math>\textbf{(A) } \text{Bela will always win.}</math> <math>\textbf{(B) } \text{Jenn will always win.}</math> <math>\textbf{(C) } \text{Bela will win if and only if }n \text{ is odd.}</math> <math>\textbf{(D) } \text{Jenn will win if and only if }n \text{ is odd.}</math> <math>\textbf{(E) } \text{Jenn will win if and only if }n > 8.</math>
 
 
 
==Solution==
 
 
 
We can see that if Bela chooses <math>\frac{n}{2}</math>, she splits the line into two halves. After this, she can simply mirror Jenn's moves, and because she now goes after Jenn, Bela will always win. Thus, our answer is <math>\boxed{\textbf{(A) } \text{Bela will always win.}}</math>
 
 
 
==See Also==
 
 
 
{{AMC12 box|year=2020|ab=B|num-b=13|num-a=15}}
 
{{MAA Notice}}
 

Latest revision as of 16:37, 8 February 2020